1answer.
Ask question
Login Signup
Ask question
All categories
  • English
  • Mathematics
  • Social Studies
  • Business
  • History
  • Health
  • Geography
  • Biology
  • Physics
  • Chemistry
  • Computers and Technology
  • Arts
  • World Languages
  • Spanish
  • French
  • German
  • Advanced Placement (AP)
  • SAT
  • Medicine
  • Law
  • Engineering
IceJOKER [234]
3 years ago
13

A stock has a beta of 1.15, the expected return on the market is 10.3 percent, and the risk-free rate is 3.1 percent. What must

the expected return on this stock be?
Business
1 answer:
kvv77 [185]3 years ago
8 0

Answer:

The expected return on this stock is 11.38%.

Explanation:

We apply the Capital Asset Pricing Model (CAPM) to solve the problem.

Under the CAPM, we have:

Return on a stock = Risk-free rate + Beta * ( Return on Market - Risk free rate).

in which:

Risk-free rate is given at 3.1%;

Beta is given at 1.15;

Return on Market is given at 10.3%;

So:

Return on a stock = Risk-free rate + Beta * ( Return on Market - Risk free rate) = 3.1% + 1.15 * ( 10.3% - 3.1%) = 11.38%.

Thus, the answer is 11.38%.

You might be interested in
Strieby Corp. operates a business with three separate entities as subsidiaries that operate as a unitary business: Matty Corp.,
Zielflug [23.3K]

Answer:

Those that will be required to file a California return are; Matty,

Strieby, Kennedy and Jackson .

Explanation:

Those that will be required to file a California return are; Matty, Strieby, Kennedy and Jackson. From the question we are informed that all of the entities involved are operating unitary Business, which is the same concept followed by California. And we know that

Unitary business can be regarded as group of persons which are related and having interdependent business activities/operations. So in this case, single California return will be filled by the entities involved.

3 0
3 years ago
What did Lonnie Johnson work on in the military
arlik [135]

Lonnie Johnson had an illustrious career as a nuclear engineer working on the Galileo space programme and the stealth bomber - but he is best known for creating a water pistol.

Hope this helped!

Good luck :p

~Emmy <3

5 0
3 years ago
Customer Information Programs (CIPs) use several examples to define a person opening a new account. In which scenario should the
xz_007 [3.2K]

Answer:

Option (D) is the right answer.

Explanation:

According to the scenario, the most appropriate answer is option ( D) because Stanley smith is a customer of the bank as he has a checking account in the bank.

While the other options are wrong because of the following reasons:

  • Option (A) is incorrect because the company lucky licks Inc. is the customer of the bank, not any person.
  • Option (B) is incorrect because Cara is not the customer of the bank as she only uses the bank's ATM.
  • Option (C) is incorrect because Herman is not the customer of the bank as he only cashes his checks in the bank.
8 0
3 years ago
Suppose in the short run a firm’s production function is given by Q = L 1 2 K 1 2 and that K is fixed at K = 10. If the price of
Furkat [3]

The firm’s marginal cost of production when the firm is producing 50 units of output is 33.33

Solution:

The production function is Q = \sqrt{L * K}

The initial value is 10 units. The production value is 50 units The manufacturing cycle needs work as stated below.

Q = \sqrt{L * K}

Q = \sqrt{L * 10}

L = (\frac{Q}{3.162} )^{2}

The wage rate is $15 . The following is the expense of the manufacturing process.

TC = P_{L} * L + P_{K} * K

TC = ( 15 * (\frac{Q}{3.162} )^{2} ) + [ P_{k * 10}]

The marginal production cost is really the increase in manufacturing costs as output increases by 1 point.

As listed below, the marginal cost:

TC = ( 15 * (\frac{Q}{3.162} )^{2} ) + [ P_{k * 10}]

MC = \frac{TC}{Q} = \frac{2Q}{3}

MC = \frac{2*50}{3} = 33.33

6 0
3 years ago
Bo Borg is the vice president of purchasing for Crater Corp. He has authority to enter into purchase contracts on behalf of Crat
g100num [7]

Answer: Crater will be bound because of Borg's apparent authority.

Explanation:

Crater Corp. will be bound to the contract since Bo Borg has the apparent authority as the acting Vice President of purchasing. Even though he went over the agreed amount that was over 2 million in the contract. Since the Shady company was unaware that he had exceeded his authority the contract will stay in place. If Shady company had of known that he did not have the final say and needed approval the result of the transaction would of been different.

7 0
3 years ago
Other questions:
  • Rose decides she wants to begin her own business, marketing toward online game players. Which of the following best answers the
    15·1 answer
  • Mary wants to purchase a 20-year bond that has a par value of $1,000 and makes semiannual interest payments of $40. If her requi
    9·1 answer
  • g Two companies market new batteries targeted at owners of personal music players. Dura Tunes claims a mean battery life of 11 h
    12·1 answer
  • Miel Company produces ready-to-cook oatmeal. Each carton of oatmeal requires 16 ounces of rolled oats per carton (the unit quant
    13·1 answer
  • If your business operates in more than one state or in a large area, it's a good idea to register the trademark with what organi
    13·1 answer
  • Suppose that supply changes such that at each price, 20 fewer towels are offered for sale. In other words, the new supply is now
    10·1 answer
  • The result of market research are used to identify the for a product
    10·1 answer
  • Discretionary fiscal policy is defined as fiscal policy Group of answer choices initiated by a Presidential proclamation. left t
    6·1 answer
  • Jessica submitted to cancel event request form to cancel event she is scheduled to conduct three weeks from today the only adver
    6·1 answer
  • A monopolist maximizes profits at the output at which Group of answer choices total revenue is at its greatest, assuming that th
    11·2 answers
Add answer
Login
Not registered? Fast signup
Signup
Login Signup
Ask question!